Частица в односторонней бесконечной ступенчатой ​​функции плюс притягивающая яма с дельта-функцией [закрыто]

Учитывая следующий потенциал:

{ Икс г В 0 дельта ( Икс ) Икс > г
при d > 0

Я хотел бы вычислить условие, которое необходимо проверить, чтобы иметь хотя бы одно связанное состояние.

От ЧАС ^ ψ ( Икс ) "=" Е н ψ ( Икс ) и предполагая, что у нас есть связанное состояние (E < 0), я получаю общее решение для ψ ( Икс ) :

{ 0 Икс < г А ( е 2 к г е к Икс + е к Икс ) г < Икс < 0 С е к Икс Икс > 0

где к "=" 2 м | Е | 2

Накладывая граничные условия в Икс "=" 0 : Δ г ψ ( Икс ) г Икс "=" 2 м В о 2 ψ ( 0 ) , ψ ( 0 ) "=" ψ ( 0 + ) и ψ ( г ) "=" 0 ,

Я заключаю, что квантование энергии системы определяется выражением:

1 + е 2 к г 1 е 2 к г "=" 1 2 м В 0 2 к
Что не имеет никакого смысла из-за того, что к г > 0 и левый член этого уравнения больше 1 в этой области.

Может ли кто-нибудь объяснить мне, что не так?

В Икс "=" г , твоя середина ψ оценивает А ( е 3 к г + е + к г ) 0 .

Ответы (3)

Распределение дельты позиции дельта ( Икс ) имеет единицы обратной длины, так что г Икс   дельта ( Икс ) "=" 1 является размерно-согласованным. Таким образом, размерная согласованность предполагает, что ваш потенциал должен быть

В ( Икс ) "=" { Икс < г В 0 а дельта ( Икс ) Икс > г

где а новый параметр с единицами измерения длины.

В пределе г вы ожидаете восстановить единственное связанное состояние изолированного дельта-потенциала; дополнительный параметр предполагает, что вы должны восстановить связанное состояние в пределе г а .

Обратите внимание, что в вашем условии квантования 2 м В 0 2 к имеет те же единицы, что и к , но вы используете его в безразмерном контексте.

Тем не менее, изменение В 0 В 0 а не устраняет проблему, которую вы определили в своем условии квантования, что одна сторона определенно больше, чем другая, а другая сторона определенно меньше. Проблема все еще присутствует, даже если вы принимаете г ограничить и написать

лим г 1 + е 2 к г 1 е 2 к г "=" 1 "=" ? ! 1 2 м В 0 а 2 к

Есть два возможных объяснения этого сценария:

  1. Вы допустили ошибку в настройке граничных условий. Когда вы исправите эту ошибку, вы обнаружите известное решение Е "=" м ( В 0 а ) 2 / 2 2 для г лимит и коррекция порядка а / г за далеким бесконечным барьером.

  2. Связанное состояние дельта-потенциала настолько «тонкое», что бесконечный потенциальный барьер на любом расстоянии разрушает его. В этом случае вы можете посмотреть на конечный барьер

    В ( Икс ) "=" { В 1 Икс < г В 0 а дельта ( Икс ) Икс > г
    и ожидать, что существование связанного состояния будет иметь такое условие, как В 1 г В 0 а .

Я склоняюсь к № 1, еще одной ошибке, но я бы не слишком удивился в любом случае.


В вашем среднем определении слова есть ошибка со знаком ψ :

ψ | Икс "=" г "=" А ( е 3 к г + е к г ) 0

Использование вместо

ψ | г < Икс < 0 "=" А ( е + 2 к г е + к Икс + е к Икс )

дает условие квантования

к "=" м В 0 а 2 ( 1 е 2 к г )

Для г это на самом деле сводится к условию невозмущенной притягивающей дельта-ямы, и мы восстанавливаем единственное решение к "=" м В 0 а 2 . Для конечных г , есть решение с к "=" 0 что соответствует (после нормализации) нулевой всюду волновой функции. Нетривиальное решение существует только в том случае, если правая часть изначально круче к чем левая сторона; то есть, если

2 г > 1 к

Значение решения будет включать W-функцию Ламберта (нормальный человек найдет ее численно). Удовлетворительно, это тот случай, когда «более глубокая» или «более широкая» привлекательная скважина с большей В 0 или а , с большей вероятностью сохранит свое связанное состояние при заданном г . Моя интуиция о разделении «силы» скважины на два фактора с интерпретируемыми единицами измерения в данном случае оказалась бесполезной.

Большое спасибо! Не могли бы вы объяснить, как получить это выражение для квантования энергии?
Так же, как и рецепт, который вы выложили. Фиксация ψ ( г ) "=" 0 дает е + 2 к г фактор в волновой функции слева от нуля; фиксация ψ ( 0 ) "=" ψ ( 0 + ) дает отношение между А и С ; фиксация Δ г ψ г Икс "=" 2 м В 0 а 2 ψ ( 0 ) упрощается до выражения, которое я дал, которое также (перестроено для В 0 ) в ответе Ч.Р. Дроста. Я предпочитаю мое расположение, потому что вы можете зарисовать как левую, так и правую сторону как функции к и посмотреть качественно, сколько пересечений они будут иметь. Я использовал систему компьютерной алгебры, чтобы отслеживать детали.
Я понимаю!! большое спасибо

Будьте осторожны с этими дельтами

Итак, в первой части вашего вопроса вы, кажется, правильно определяете форму

ψ ( Икс ) "=" А ( е к | Икс | е к ( Икс + 2 г ) ) ,
обладающий основным свойством, состоящим в том, что на каждом интервале он имеет А е к Икс + Б е к Икс зависимость, и она должна стремиться к нулю при Икс "=" г и существует непрерывность значения, но разрыв производной в точке Икс "=" 0.

Тогда у вас есть желание установить к чтобы «изгиб» волновой функции соответствовал потенциалу дельта-функции с помощью уравнения Шредингера,

Е ψ ( Икс ) "=" 2 2 м ψ ( Икс ) В 0 дельта ( Икс ) ψ ( Икс )
(Одно предостережение, так же как и потенциал дельта-функции Дирака, единицы В 0 являются джоуль-метрами.) Таким образом, мы интегрируем обе части в небольшом интервале ϵ ϵ г Икс и получить ваше отношение
Е ψ ( 0 ) 2 ϵ "=" 2 2 м [ ψ ( ϵ ) ψ ( ϵ ) ] В 0 ψ ( 0 ) .
Конечно, в пределе левая часть стремится к нулю, и мы получаем ваше «граничное условие», но, может быть, позвольте мне немного побаловать себя и вместо того, чтобы интерпретировать это как граничное условие, давайте интерпретируем его как эксперимент по определению В 0 . Итак, кто-то дает нам эту волновую функцию (скажем, с помощью томографии) и спрашивает, насколько глубоким должен быть дельта-потенциал, чтобы создать эту форму.

Сейчас второй срок ψ ( Икс ) имеет непрерывную первую производную, поэтому она не меняется между ϵ и + ϵ и поэтому он просто вычитает себя. Вместо этого мы просто смотрим на опыт ( к | Икс | ) который резко переходит от склона + к наклоняться к при х=0,

0 "=" 2 2 м [ к А к А ] В 0   А ( 1 е 2 к г ) , В 0 "=" 2 к м ( 1 е 2 к г ) .
Так что это выглядит хорошо и легко вычисляется.

Вот где я думаю, что вы могли пойти не так? Не поймите меня неправильно, мне нравится ваш числитель 1 е 2 к г больше, похоже, я мог бы превратить его в гиперболический тангенс, может быть, вывести его проще, переведя Икс Икс + г или так, чтобы на интервале 0 < Икс < г затем ф ( Икс ) "=" А грех ( к Икс ) , ф ( Икс ) "=" А к чушь ( к Икс ) , бам, бам, сделано как раз к чаю. Но я думаю, что ты только что сделал ф ( 0 ) / ф ( 0 ) или так и это просто не работает таким образом? Не уверен.

простите меня, я сам себя ботан

Теперь у нас также есть интересная проблема, которая заключается в том, что мы, по-видимому, зависим от этой функции. ф ( Икс ) "=" Икс / ( 1 е Икс ) быть обратимым, если мы хотим идти в том направлении, в котором вы хотите идти, как

к "=" 1 2 г   ф 1 ( 2 г м В 0 2 ) ,
И тогда вы можете получить кинетическую энергию с помощью 2 к 2 / 2 м по обыкновению. Теперь мы знаем, что ф не будет иметь обратного с точки зрения каких-либо элементарных функций , но реальная опасность заключается в том, что у него вообще не будет обратного . Если ф не может иметь обратного тогда, учитывая некоторые В 0 нет таких к .

Один простой способ показать, что что-то обратимо, — это показать, что оно монотонно возрастает (достаточно, но не обязательно), и я думаю, мы можем сделать это здесь? Производная, по-видимому, имеет положительный квадрат знаменателя и числитель, который можно разложить как е Икс ( е Икс Икс 1 ) и разложение Тейлора второго е Икс делает это также произведением двух положительных чисел... Итак, если производная всегда положительна, то она монотонно возрастает и, следовательно, обратима.

На самом деле Wolfram Alpha даже лучше и говорит нам, что обратное просто ф 1 ( Икс ) "=" Икс + Вт ( Икс е Икс ) , где Вт это «функция журнала продукта».

Так что я думаю, вам все же может понадобиться условие адекватности для физики, наверное к > 0 так что ψ ( Икс ) нормируется на положительную бесконечность. К счастью, это просто ф ( 0 ) который мы можем легко вычислить, и, к счастью, единственное условие, которое нам нужно, это В 0 > 0 чему я немного удивлен и благодарен.

Большое спасибо за ответ!! Вы заставляете меня осознать многие вещи, которые я не принял во внимание!! Я буду следовать вашим советам.
Когда вы говорите об условии адекватности, не могли бы вы объяснить мне, в чем смысл вашего утверждения, что k > 0? Для получения общего вида ψ ( Икс ) для проблемы я предположил это, поэтому я выбрал Exp[-kx] вместо Exp[kx], когда x > 0. Большое вам спасибо!!
Да, именно. Этот выбор работает только до тех пор, пока к > 0 .

Мое предположение состояло бы в том, чтобы сказать, что вы правы, что означало бы, что для этого потенциала нет связанных состояний. Я думаю, что это так, поскольку для простой привлекательной дельта-функции существует только одно связанное состояние, поэтому, если мы добавим еще одно условие (в данном случае бесконечный шаг), это может удалить это уникальное разрешенное состояние, что приведет к недопустимому состоянию. связанное состояние.

РЕДАКТИРОВАТЬ: Если в вашем упражнении указано, что существует связанное состояние, мое предположение должно быть неверным, но я все еще не вижу ошибки в ваших расчетах...

Ваше объяснение имеет смысл для меня. Что меня смущает, так это то, что в упражнении говорится, что существует связанное состояние. Спасибо!
Это должно быть физически неправильно. Существует связанное состояние для потенциала дельта-функции само по себе, и было бы шокирующим, если бы оно исчезло, если бы вы добавили потенциальную границу на бесконечности. Более того, ОП рассчитал для вас волновую функцию, А ( е к | Икс | е к ( Икс + 2 г ) ) и, кажется, мало что может исключить его по физическим причинам, его можно нормализовать и т. д. ... Единственный заданный интересный вопрос заключается в том, как установить разрыв в первой производной как конкретное значение и как прочитать результирующее собственное значение энергии ... «нет никакого способа настраивать к чтобы получить определенный разрыв» звучит неправильно.